LSAT and Law School Admissions Forum

Get expert LSAT preparation and law school admissions advice from PowerScore Test Preparation.

User avatar
 Dave Killoran
PowerScore Staff
  • PowerScore Staff
  • Posts: 5852
  • Joined: Mar 25, 2011
|
#79628
Complete Question Explanation
(The complete setup for this game can be found here: lsat/viewtopic.php?t=6157)

The correct answer choice is (B).

If L is connected to M and O, the following connections result:

PT6-Oct 1992 LG Explanations game 4 #23 diagram 1.png
With L connected to O, O is closed and thus from our inference discussion K must connect to M:
PT6-Oct 1992 LG Explanations game 4 #23 diagram 2.png
Remember, because the bridges can curve as much as needed, we are not worried about the apparent intersection above. We would easily overcome that problem by looping a bridge from K to M around the other islands, but that is a waste of time.

Accordingly, answer choice (B) is correct.

Note that we can also infer that J must connect to L, because J must connect to exactly two bridges, and K, M, and O all have the maximum number of connections allowed by the rules. This information eliminates answer choices (A) and (D).
You do not have the required permissions to view the files attached to this post.
User avatar
 samanthaolive
  • Posts: 1
  • Joined: Jun 21, 2021
|
#88148
I'm confused about this question due to one of the inferences made:

(L----O) ----> (K---M)
(K---O)---->(L----M)

By connecting K and M, I would think this breaks one of the conditions therefore, that forced me to deny that answer choice. Would you mind explaining this to me, please?
User avatar
 Ryan Twomey
PowerScore Staff
  • PowerScore Staff
  • Posts: 141
  • Joined: Mar 04, 2021
|
#88180
Hey Samantha Olive,

I would need you to specify which condition the K with M condition breaks. There are no rules that K with M breaks.

But here is how I diagrammed this question, step by step.

1. I brought over the J-----O and the M----O from my original rules. (Rule #7)

2. Then I brought over the L-----M and the L-----O from this question.

3. Then I realized that now we have 3 islands connecting with O, so nobody else can connect with O (because of rule 3)

4. Then I looked at rule 4 (J,K and L are each directly connected by bridge with M or O or both) I then determined that O could not have anymore people connecting to it because of my step #3, so then I realized that K has to connect with M.

5. Because this is a must be true question, I stopped trying to make any more deductions and checked to see if my deduction that K connects by bridge with M was an answer choice. It was answer choice B, so I selected it and moved on to the next question.

I hope my step by step explanation helps. I thought Dave's drawings in this game was very helpful, and I wish you all of the luck in your studies.

Best,
Ryan
User avatar
 Bmas123
  • Posts: 22
  • Joined: Aug 24, 2022
|
#99052
Hi! Why could this not have been C? If K was connected to O, it would not have to be connected to M. We could stil have J connected to O and J could have connected to either J or L to complete the 2nd line J needed. L could connect to M to satisfy that condition. What am I missing as to why this is incorrect? Thank you!
User avatar
 Bmas123
  • Posts: 22
  • Joined: Aug 24, 2022
|
#99053
Bmas123 wrote: Sun Jan 29, 2023 9:24 pm Hi! Why could this not have been C? If K was connected to O, it would not have to be connected to M. We could stil have J connected to O and J could have connected to either J or L to complete the 2nd line J needed. L could connect to M to satisfy that condition. What am I missing as to why this is incorrect? Thank you!
I think its the wording of the actual question that is confusing me, but I cant figure out exactly what it is saying!
 Luke Haqq
PowerScore Staff
  • PowerScore Staff
  • Posts: 742
  • Joined: Apr 26, 2012
|
#99083
Hi Bmas123!

This question asks for what must be true given the stated conditions, and answer choice (C) states, "A bridge directly connects K with O."

Answer choice (C) is incorrect because it cannot be true. This is because of the rule, "No island has more than three bridges that directly connect it with other islands."

The last rule already connects O with two other islands--J and M. Then in this question O is also connected with L. So under those conditions, O is connected with three islands--J, M, and L. O therefore cannot also be connected with K, since this would connect it to four islands, in violation of the rule that no island has more than three bridges.

Get the most out of your LSAT Prep Plus subscription.

Analyze and track your performance with our Testing and Analytics Package.